Old Wall Quiz #6 Flashcards

1
Q
  1. Which of the following is the most likely an
    adverse effect of bisphosphonate therapy?
    A. Diarrhea
    B. Gastrointestinal ulceration
    C. Nausea and Vomiting
    D. Deep Venous thrombosis
    E. Osteonecrosis of the Jaw
A

b

How well did you know this?
1
Not at all
2
3
4
5
Perfectly
2
Q
  1. A 22 y/o gravida at 32 weeks is noted to be
    somnolent, her pupils are pinpoint, and she
    often misses her prenatal appointments. What
    is the most likely diagnosis?
    A. Alcohol abuse
    B. Cocaine abuse
    C. Methamphetamine abuse
    D. Opioid abuse
    E. Normal pregnancy
A

d

How well did you know this?
1
Not at all
2
3
4
5
Perfectly
3
Q
3. Which of the following medications is NOT a
cause of hirsutism or hypertrichosis?
A. Danazol
B. Minoxidil
C. Methyldopa
D. Ketoconazole
E. Corticosteroids
A

d

How well did you know this?
1
Not at all
2
3
4
5
Perfectly
4
Q
4. A 24 year old presents to your office. On
breast exam you palpate a firm nodular breast
mass. What is the next step?
A. Ultrasound
B. Core needle biopsy
C. Wide local excision
D. Diagnostic Mammography
E. Ductography
A

a

How well did you know this?
1
Not at all
2
3
4
5
Perfectly
5
Q
  1. What is a first-line controller therapy for
    persistent asthma during pregnancy?
A. Albuterol MDI
B. Albuterol nebulized
C. Humidifier
D. Low dose inhaled steroids
E. Antibiotics
A

d

How well did you know this?
1
Not at all
2
3
4
5
Perfectly
6
Q
  1. Which of the following diagnostic studies are
    warranted to confirm a bowel obstruction?
    A. Flat plate and upright
    B. CT with contrast
    C. MRI
    D. Barium Enema
    E. Upper GI series
A

b

How well did you know this?
1
Not at all
2
3
4
5
Perfectly
7
Q
8. A 2 year old child with labial adhesion and
difficulty urinating presents to your office. What
is the best management?
A. Estrogen Cream
B. Surgical Repair
C. Testosterone cream
D. Labial separation
E. Corticosteroids
A

a

How well did you know this?
1
Not at all
2
3
4
5
Perfectly
8
Q
  1. A 22 year-old G1P0 at 6 weeks gestation is
    exposed to an estimated dose of 250 mGy of
    radiation, what is the likely teratogenic effect?
    A. Growth restriction
    B. Microcephaly
    C. Intellectual disability
    D. Limb hypoplasia
    E. Death of embryo
A

a

How well did you know this?
1
Not at all
2
3
4
5
Perfectly
9
Q
10. What HPV serotype is the most common
type present in 55% of vaginal cancer
samplings?
A. 16
B. 18
C. 31
D. 33
E. 45
A

a

How well did you know this?
1
Not at all
2
3
4
5
Perfectly
10
Q
12. Which of the following cancers is least likely
associated with Lynch II (HNPCC)?
A. Brain
B. Ovarian
C. Small Bowel
D. Cervical
E. Endometrial
A

d

How well did you know this?
1
Not at all
2
3
4
5
Perfectly
11
Q
  1. A 44 year old presents with a TSH of 14
    Patient is started on 100 mcgs of thyroxine. Two
    months later her repeat TSH is now 7. What is
    the next step in management?
    A. Repeat TSH in 3 months
    B. Continue same dose of Thyroxine
    C. Add Polypropylurical (PTU)
    D. Increase Thyroxine dose
    E. Add Methimazole
A

d

How well did you know this?
1
Not at all
2
3
4
5
Perfectly
12
Q
  1. A patient of yours has been involved in series
    of domestic disputes with her partner. On
    evaluation, all of the following should be part of
    the conversation EXCEPT:
    A. Assess the immediate safety of the patient
    B. Report any abuse or neglect to the children
    C. Patient made aware of help hotlines
    D. Assistance is offered to her to develop a
    escape plan
    E. Call state attorney to immediately enforce an
    order of protection
A

e

How well did you know this?
1
Not at all
2
3
4
5
Perfectly
13
Q
15. A 65 year old with distal interphalangeal
joint pain and swelling, which of the following is
the recommended first line therapy?
A . Methotrexate
B. Acetaminophen
C. Opioids
D. Steroids
E. Tofacitinib (Xeljanz)
A

B

How well did you know this?
1
Not at all
2
3
4
5
Perfectly
14
Q
17. A 45 year old patient presents to your office
with a blood pressure 160/104, and is
asymptomatic, what is the next best step?
A. Start Thiazide diuretic
B. Repeat Blood pressure
C. Start Beta blocker
D. Electrocardiogram
E. Lifestyle modifications
A

b

How well did you know this?
1
Not at all
2
3
4
5
Perfectly
15
Q
  1. A neonate was delivered and was found to
    have fused labioscrotal folds. What is the
    inheritance pattern?
    A. Autosomal recessive
    B. X linked recessive
    C. Autosomal dominant
    D. Multifactorial
A

a

How well did you know this?
1
Not at all
2
3
4
5
Perfectly
16
Q
23. Which of the following is least likely
associated with Androgen insensitivity
syndrome?
A. Presents with primary amenorrhea
B. Karyotype is 46xy
C. Testosterone in male level
D. X-Linked recessive inheritance
E. Male phenotype
A

e

17
Q
  1. A 75 y/o s/p TAHBSO presents with POP-Q
    measurements: Aa:+3, Ba:+8, Bp:=+8, C=+8,
    Ap=+3. She is sexually active and requests
    definitive management. Which of the
    following is the best option for her?
    A. Lefort procedure
    B. Colpocleisis
    C. Abdominal sacral colpopexy
    D. Kegel’s exercises
    E. Estrogen cream
A

c

18
Q
  1. A 48 year old presents for her cervical
    screening cytology results. Cytology and HPV negative, atypical endometrial cells present.
    What is the next best step?
A. Excisional procedure
B. Endocervical and Endometrial sampling
C. Repeat cytology and HPV testing in 6 months
D. Routine screening
E. Repeat cytology and HPV testing in 12
months
A

b

19
Q
29. A patient presents with foot drop after a
Laparoscopic vaginal hysterectomy. Which nerve
was affected?
A. Femoral
B. Tibial
C. Lateral femoral cutaneous
D. Obturator
E. Peroneal
A

e

20
Q
32. Which of the following birth control
methods has the highest failure rate?
A. Implantable progestin
B. Withdrawal
C. Levonorgestrel IUD
D. Oral contraceptives
E. Spermicide
A

e

21
Q
34. Which percentage of fetuses are breech at
28 weeks?
A. 1%
B. 5%
C. 10%
D. 25%
E. 50%
A

d

22
Q
  1. Which of the following patients is the best
    candidate for a uterine artery
    embolization(UAE)?
    A. Pedunculated Fibroid
    B. Submucosal fibroid
    C. Cervical fibroid protruding through cervical os
    D. Enlarged 15 week size uterus with intramural
    fibroids
    E. Degenerating fibroid
A

d

23
Q
39. Which of the following percentage is the
upper limit of discordance recommended to
safely extract a second twin breech?
A. 1%
B. 5%
C. 10%
D. 15%
E. 20%
A

e

24
Q
  1. After the age of 37, which of the following
    factors best explains why fecundity decreases
    with age:
    A. Quality of oocytes
    B. Quantity of oocytes
    C. Irregular menstrual cycles/anovulation
    D. Decrease in motility of spermatozoa
    E. Loss of endometrial support
A

a

25
Q
47. All of the following can potentially cause
uterine tachysystole EXCEPT:
A. Dinoprostone
B. Misoprostol
C. Amniotomy
D. Pitocin
E. Breast stimulation
A

c